2018 AMC 12A Problem #19

Algebra Level 3

Let A A be the set of positive integers that have no prime factors other than 2 , 2, 3 , 3, or 5. 5. The infinite sum 1 1 + 1 2 + 1 3 + 1 4 + 1 5 + 1 6 + 1 8 + 1 9 + 1 10 + 1 12 + 1 15 + 1 16 + 1 18 + 1 20 + \dfrac{1}{1} + \dfrac{1}{2} + \dfrac{1}{3} + \dfrac{1}{4} + \dfrac{1}{5} + \dfrac{1}{6} + \dfrac{1}{8} + \dfrac{1}{9} + \dfrac{1}{10} + \dfrac{1}{12} + \dfrac{1}{15} + \dfrac{1}{16} + \dfrac{1}{18} + \dfrac{1}{20} + \cdots of the reciprocals of the elements of A A can be expressed as m n , \frac{m}{n}, where m m and n n are relatively prime positive integers. What is m + n m + n ?


Next problem

Previous problem

All problems

15 23 17 36 19

This section requires Javascript.
You are seeing this because something didn't load right. We suggest you, (a) try refreshing the page, (b) enabling javascript if it is disabled on your browser and, finally, (c) loading the non-javascript version of this page . We're sorry about the hassle.

1 solution

We can factor the given infinite sum as

( 1 + 1 2 + 1 2 2 + 1 2 3 + . . . . ) ( 1 + 1 3 + 1 3 2 + 1 3 3 + . . . . ) ( 1 + 1 5 + 1 5 2 + 1 5 3 + . . . . ) = 1 1 1 2 × 1 1 1 3 × 1 1 1 5 = 2 × 3 2 × 5 4 = 15 4 \left(1 + \dfrac{1}{2} + \dfrac{1}{2^{2}} + \dfrac{1}{2^{3}} + .... \right) \left(1 + \dfrac{1}{3} + \dfrac{1}{3^{2}} + \dfrac{1}{3^{3}} + .... \right) \left(1 + \dfrac{1}{5} + \dfrac{1}{5^{2}} + \dfrac{1}{5^{3}} + .... \right)\ = \dfrac{1}{1 - \frac{1}{2}} \times \dfrac{1}{1 - \frac{1}{3}} \times \dfrac{1}{1 - \frac{1}{5}} = 2 \times \dfrac{3}{2} \times \dfrac{5}{4} = \dfrac{15}{4} .

Thus m + n = 15 + 4 = 19 m + n = 15 + 4 = \boxed{19} .

0 pending reports

×

Problem Loading...

Note Loading...

Set Loading...